Existence of limit for every adherence point in a bounded set $U$ implies $f(U)$ is bounded The 2019 Stack Overflow Developer Survey Results Are InExistence of mixed partials in Clairaut's theorem.Every continuous function $f: mathbbR to mathbbR$ is uniformly continuous on every bounded set.Not equicontinuoushow to determine the existence of double limit?Every point in the open set is a limit pointDerivative and uniform continuityConvergent subsequence for every sequence implies complete and totally boundedSet representation of bounded functionsIf omega limit set contains only one point, $x^*$, then $lim_ttoinftyphi(t;x_0)=x^*$Proving a limit diverges if $f$ is not bounded above

A word that means fill it to the required quantity

Slides for 30 min~1 hr Skype tenure track application interview

What is the most efficient way to store a numeric range?

Did Scotland spend $250,000 for the slogan "Welcome to Scotland"?

What do I do when my TA workload is more than expected?

Can a rogue use sneak attack with weapons that have the thrown property even if they are not thrown?

"as much details as you can remember"

Correct punctuation for showing a character's confusion

Keeping a retro style to sci-fi spaceships?

Is it okay to consider publishing in my first year of PhD?

How can I define good in a religion that claims no moral authority?

Are turbopumps lubricated?

If I can cast sorceries at instant speed, can I use sorcery-speed activated abilities at instant speed?

ODD NUMBER in Cognitive Linguistics of WILLIAM CROFT and D. ALAN CRUSE

What could be the right powersource for 15 seconds lifespan disposable giant chainsaw?

Mathematics of imaging the black hole

How to charge AirPods to keep battery healthy?

Is it possible for absolutely everyone to attain enlightenment?

How do PCB vias affect signal quality?

Deal with toxic manager when you can't quit

How do I free up internal storage if I don't have any apps downloaded?

Accepted by European university, rejected by all American ones I applied to? Possible reasons?

Match Roman Numerals

Is it safe to harvest rainwater that fell on solar panels?



Existence of limit for every adherence point in a bounded set $U$ implies $f(U)$ is bounded



The 2019 Stack Overflow Developer Survey Results Are InExistence of mixed partials in Clairaut's theorem.Every continuous function $f: mathbbR to mathbbR$ is uniformly continuous on every bounded set.Not equicontinuoushow to determine the existence of double limit?Every point in the open set is a limit pointDerivative and uniform continuityConvergent subsequence for every sequence implies complete and totally boundedSet representation of bounded functionsIf omega limit set contains only one point, $x^*$, then $lim_ttoinftyphi(t;x_0)=x^*$Proving a limit diverges if $f$ is not bounded above










0












$begingroup$


Let $U subseteq R^n$ be a bounded set, $f:U rightarrow mathbbR$ and
suppose that for every $x_0 in overline U$, $displaystylelim_x rightarrow x_0f(x)$ exist.
Prove $f(U)$ is bounded.



By the hypothesis, if $x_0 in overline U$ , then for every $epsilon >0$ there exists $delta>0$ such that if $left | x-x_0 right |<delta$ then $left | f(x)-L right |<epsilon$. I don't really see how I can obtain a constant $M>0$ such that $left | w-z right |<M$ for every $w,z in f(U)$. If $f$ was uniformly continuous I can see why $f(U)$ would be bounded but with the limit hypothesis only I'm stuck. Any guideline is welcome.










share|cite|improve this question











$endgroup$
















    0












    $begingroup$


    Let $U subseteq R^n$ be a bounded set, $f:U rightarrow mathbbR$ and
    suppose that for every $x_0 in overline U$, $displaystylelim_x rightarrow x_0f(x)$ exist.
    Prove $f(U)$ is bounded.



    By the hypothesis, if $x_0 in overline U$ , then for every $epsilon >0$ there exists $delta>0$ such that if $left | x-x_0 right |<delta$ then $left | f(x)-L right |<epsilon$. I don't really see how I can obtain a constant $M>0$ such that $left | w-z right |<M$ for every $w,z in f(U)$. If $f$ was uniformly continuous I can see why $f(U)$ would be bounded but with the limit hypothesis only I'm stuck. Any guideline is welcome.










    share|cite|improve this question











    $endgroup$














      0












      0








      0





      $begingroup$


      Let $U subseteq R^n$ be a bounded set, $f:U rightarrow mathbbR$ and
      suppose that for every $x_0 in overline U$, $displaystylelim_x rightarrow x_0f(x)$ exist.
      Prove $f(U)$ is bounded.



      By the hypothesis, if $x_0 in overline U$ , then for every $epsilon >0$ there exists $delta>0$ such that if $left | x-x_0 right |<delta$ then $left | f(x)-L right |<epsilon$. I don't really see how I can obtain a constant $M>0$ such that $left | w-z right |<M$ for every $w,z in f(U)$. If $f$ was uniformly continuous I can see why $f(U)$ would be bounded but with the limit hypothesis only I'm stuck. Any guideline is welcome.










      share|cite|improve this question











      $endgroup$




      Let $U subseteq R^n$ be a bounded set, $f:U rightarrow mathbbR$ and
      suppose that for every $x_0 in overline U$, $displaystylelim_x rightarrow x_0f(x)$ exist.
      Prove $f(U)$ is bounded.



      By the hypothesis, if $x_0 in overline U$ , then for every $epsilon >0$ there exists $delta>0$ such that if $left | x-x_0 right |<delta$ then $left | f(x)-L right |<epsilon$. I don't really see how I can obtain a constant $M>0$ such that $left | w-z right |<M$ for every $w,z in f(U)$. If $f$ was uniformly continuous I can see why $f(U)$ would be bounded but with the limit hypothesis only I'm stuck. Any guideline is welcome.







      real-analysis multivariable-calculus






      share|cite|improve this question















      share|cite|improve this question













      share|cite|improve this question




      share|cite|improve this question








      edited Mar 30 at 21:45









      Aloizio Macedo

      23.8k24088




      23.8k24088










      asked Mar 30 at 21:36









      ipreferpiipreferpi

      388




      388




















          1 Answer
          1






          active

          oldest

          votes


















          2












          $begingroup$

          Suppose that $f(U)$ is not bounded. Then there exists a sequence $(x_n)$ in $U$ such that $f(x_n)$ diverges to infinity ($+infty$ or $-infty$).



          Now $U$ is bounded, so there exists a subsequence $(x_varphi(n))$ of $(x_n)$ converging to a point $y in overlineU$. By hypothesis, $lim_x rightarrow y f(x)$ exists, so $lim_n rightarrow +infty f(x_varphi(n))$ exists. It is impossible because by construction, $f(x_varphi(n))$ has to diverge to infinity.






          share|cite|improve this answer









          $endgroup$












          • $begingroup$
            By contradiction seems the hypothesis works very well. Thanks.
            $endgroup$
            – ipreferpi
            Mar 30 at 21:53











          Your Answer





          StackExchange.ifUsing("editor", function ()
          return StackExchange.using("mathjaxEditing", function ()
          StackExchange.MarkdownEditor.creationCallbacks.add(function (editor, postfix)
          StackExchange.mathjaxEditing.prepareWmdForMathJax(editor, postfix, [["$", "$"], ["\\(","\\)"]]);
          );
          );
          , "mathjax-editing");

          StackExchange.ready(function()
          var channelOptions =
          tags: "".split(" "),
          id: "69"
          ;
          initTagRenderer("".split(" "), "".split(" "), channelOptions);

          StackExchange.using("externalEditor", function()
          // Have to fire editor after snippets, if snippets enabled
          if (StackExchange.settings.snippets.snippetsEnabled)
          StackExchange.using("snippets", function()
          createEditor();
          );

          else
          createEditor();

          );

          function createEditor()
          StackExchange.prepareEditor(
          heartbeatType: 'answer',
          autoActivateHeartbeat: false,
          convertImagesToLinks: true,
          noModals: true,
          showLowRepImageUploadWarning: true,
          reputationToPostImages: 10,
          bindNavPrevention: true,
          postfix: "",
          imageUploader:
          brandingHtml: "Powered by u003ca class="icon-imgur-white" href="https://imgur.com/"u003eu003c/au003e",
          contentPolicyHtml: "User contributions licensed under u003ca href="https://creativecommons.org/licenses/by-sa/3.0/"u003ecc by-sa 3.0 with attribution requiredu003c/au003e u003ca href="https://stackoverflow.com/legal/content-policy"u003e(content policy)u003c/au003e",
          allowUrls: true
          ,
          noCode: true, onDemand: true,
          discardSelector: ".discard-answer"
          ,immediatelyShowMarkdownHelp:true
          );



          );













          draft saved

          draft discarded


















          StackExchange.ready(
          function ()
          StackExchange.openid.initPostLogin('.new-post-login', 'https%3a%2f%2fmath.stackexchange.com%2fquestions%2f3168811%2fexistence-of-limit-for-every-adherence-point-in-a-bounded-set-u-implies-fu%23new-answer', 'question_page');

          );

          Post as a guest















          Required, but never shown

























          1 Answer
          1






          active

          oldest

          votes








          1 Answer
          1






          active

          oldest

          votes









          active

          oldest

          votes






          active

          oldest

          votes









          2












          $begingroup$

          Suppose that $f(U)$ is not bounded. Then there exists a sequence $(x_n)$ in $U$ such that $f(x_n)$ diverges to infinity ($+infty$ or $-infty$).



          Now $U$ is bounded, so there exists a subsequence $(x_varphi(n))$ of $(x_n)$ converging to a point $y in overlineU$. By hypothesis, $lim_x rightarrow y f(x)$ exists, so $lim_n rightarrow +infty f(x_varphi(n))$ exists. It is impossible because by construction, $f(x_varphi(n))$ has to diverge to infinity.






          share|cite|improve this answer









          $endgroup$












          • $begingroup$
            By contradiction seems the hypothesis works very well. Thanks.
            $endgroup$
            – ipreferpi
            Mar 30 at 21:53















          2












          $begingroup$

          Suppose that $f(U)$ is not bounded. Then there exists a sequence $(x_n)$ in $U$ such that $f(x_n)$ diverges to infinity ($+infty$ or $-infty$).



          Now $U$ is bounded, so there exists a subsequence $(x_varphi(n))$ of $(x_n)$ converging to a point $y in overlineU$. By hypothesis, $lim_x rightarrow y f(x)$ exists, so $lim_n rightarrow +infty f(x_varphi(n))$ exists. It is impossible because by construction, $f(x_varphi(n))$ has to diverge to infinity.






          share|cite|improve this answer









          $endgroup$












          • $begingroup$
            By contradiction seems the hypothesis works very well. Thanks.
            $endgroup$
            – ipreferpi
            Mar 30 at 21:53













          2












          2








          2





          $begingroup$

          Suppose that $f(U)$ is not bounded. Then there exists a sequence $(x_n)$ in $U$ such that $f(x_n)$ diverges to infinity ($+infty$ or $-infty$).



          Now $U$ is bounded, so there exists a subsequence $(x_varphi(n))$ of $(x_n)$ converging to a point $y in overlineU$. By hypothesis, $lim_x rightarrow y f(x)$ exists, so $lim_n rightarrow +infty f(x_varphi(n))$ exists. It is impossible because by construction, $f(x_varphi(n))$ has to diverge to infinity.






          share|cite|improve this answer









          $endgroup$



          Suppose that $f(U)$ is not bounded. Then there exists a sequence $(x_n)$ in $U$ such that $f(x_n)$ diverges to infinity ($+infty$ or $-infty$).



          Now $U$ is bounded, so there exists a subsequence $(x_varphi(n))$ of $(x_n)$ converging to a point $y in overlineU$. By hypothesis, $lim_x rightarrow y f(x)$ exists, so $lim_n rightarrow +infty f(x_varphi(n))$ exists. It is impossible because by construction, $f(x_varphi(n))$ has to diverge to infinity.







          share|cite|improve this answer












          share|cite|improve this answer



          share|cite|improve this answer










          answered Mar 30 at 21:42









          TheSilverDoeTheSilverDoe

          5,465216




          5,465216











          • $begingroup$
            By contradiction seems the hypothesis works very well. Thanks.
            $endgroup$
            – ipreferpi
            Mar 30 at 21:53
















          • $begingroup$
            By contradiction seems the hypothesis works very well. Thanks.
            $endgroup$
            – ipreferpi
            Mar 30 at 21:53















          $begingroup$
          By contradiction seems the hypothesis works very well. Thanks.
          $endgroup$
          – ipreferpi
          Mar 30 at 21:53




          $begingroup$
          By contradiction seems the hypothesis works very well. Thanks.
          $endgroup$
          – ipreferpi
          Mar 30 at 21:53

















          draft saved

          draft discarded
















































          Thanks for contributing an answer to Mathematics Stack Exchange!


          • Please be sure to answer the question. Provide details and share your research!

          But avoid


          • Asking for help, clarification, or responding to other answers.

          • Making statements based on opinion; back them up with references or personal experience.

          Use MathJax to format equations. MathJax reference.


          To learn more, see our tips on writing great answers.




          draft saved


          draft discarded














          StackExchange.ready(
          function ()
          StackExchange.openid.initPostLogin('.new-post-login', 'https%3a%2f%2fmath.stackexchange.com%2fquestions%2f3168811%2fexistence-of-limit-for-every-adherence-point-in-a-bounded-set-u-implies-fu%23new-answer', 'question_page');

          );

          Post as a guest















          Required, but never shown





















































          Required, but never shown














          Required, but never shown












          Required, but never shown







          Required, but never shown

































          Required, but never shown














          Required, but never shown












          Required, but never shown







          Required, but never shown







          Popular posts from this blog

          Triangular numbers and gcdProving sum of a set is $0 pmod n$ if $n$ is odd, or $fracn2 pmod n$ if $n$ is even?Is greatest common divisor of two numbers really their smallest linear combination?GCD, LCM RelationshipProve a set of nonnegative integers with greatest common divisor 1 and closed under addition has all but finite many nonnegative integers.all pairs of a and b in an equation containing gcdTriangular Numbers Modulo $k$ - Hit All Values?Understanding the Existence and Uniqueness of the GCDGCD and LCM with logical symbolsThe greatest common divisor of two positive integers less than 100 is equal to 3. Their least common multiple is twelve times one of the integers.Suppose that for all integers $x$, $x|a$ and $x|b$ if and only if $x|c$. Then $c = gcd(a,b)$Which is the gcd of 2 numbers which are multiplied and the result is 600000?

          Ingelân Ynhâld Etymology | Geografy | Skiednis | Polityk en bestjoer | Ekonomy | Demografy | Kultuer | Klimaat | Sjoch ek | Keppelings om utens | Boarnen, noaten en referinsjes Navigaasjemenuwww.gov.ukOffisjele webside fan it regear fan it Feriene KeninkrykOffisjele webside fan it Britske FerkearsburoNederlânsktalige ynformaasje fan it Britske FerkearsburoOffisjele webside fan English Heritage, de organisaasje dy't him ynset foar it behâld fan it Ingelske kultuergoedYnwennertallen fan alle Britske stêden út 'e folkstelling fan 2011Notes en References, op dizze sideEngland

          Հադիս Բովանդակություն Անվանում և նշանակություն | Դասակարգում | Աղբյուրներ | Նավարկման ցանկ